AAFP q's 1

Lakukan tugas rumah & ujian kamu dengan baik sekarang menggunakan Quizwiz!

tertiary prevention examples

* targets individuals with a known disease, with the goal of limiting or preventing future complications screening diabetics for microalbuminuria, rigorous treatment of diabetes mellitus, and post-myocardial infarction prophylaxis with β-blockers and aspirin

High BP

130/80 or higher

Chronic cough is how long

8 weeks

Which one of the findings in this patient is a required diagnostic criterion for polycystic ovary syndrome? (check one) A. Hyperandrogenism B. Hypertension C. Hypothyroidism D. Infertility E. Obesity

A. Hyperandrogenism

A 67-year-old male presents to your office because of fatigue and a syncopal episode. His vital signs in the office are normal. An examination reveals a harsh systolic murmur best heard over the second right intercostal space radiating to the neck. Echocardiography confirms your suspected diagnosis.Which one of the following is the only treatment that improves mortality with this condition? (check one) A. β-Blockers B. Antimicrobial prophylaxis for bacterial endocarditis C. Aortic valve replacement D. Mitral valve repair E. Ventricular septal defect closure

C. Aortic valve replacement This patient has symptomatic severe aortic stenosis. The only treatment that improves this condition is aortic valve replacement

Which one of the following is a significant risk factor for esophageal adenocarcinoma? A. Aspirin therapy B. Ibuprofen therapy C. Helicobacter pylori infection D. Obesity E. Crohn's disease

D. Obesity

A 30-year-old female is planning a trip to Australia and is concerned about motion sickness.Which one of the following medications is most effective for preventing this problem? (check one) A. Dimenhydrinate (Dramamine) B. Diphenhydramine (Benadryl) C. Meclizine (Antivert) D. Promethazine E. Scopolamine

E. Scopolamine Motion sickness is a syndrome that includes nausea and other symptoms, including vaguesubtle symptoms of stomach awareness, malaise, fatigue, and irritability. The mosteffective medication is scopolamine; transdermal scopolamine is more effective than oralscopolamine.

which test/imaging should be ordered when PUL HTN is suspected

ECHO

Chronic orthostatic hypotension tx

Fludorocortisone , miodrine , physostigmine

Sepsis pt needs fluids , what's your drug of choice

NE

A 34-year-old female presents to your office after she was bitten on the hand by a neighbor's cat. The patient has no allergies and has been in good health. You decide to treat the patient with a prophylactic antibiotic. Which one of the following would be the antibiotic of choice? (check one) A. Amoxicillin/clavulanate (Augmentin) B. Azithromycin (Zithromax) C. Cephalexin (Keflex) D. Clindamycin (Cleocin) E. Metronidazole (Flagyl)

A. Amoxicillin/clavulanate (Augmentin)

A 34-year-old female presents to your office after she was bitten on the hand by a neighbor's cat. The patient has no allergies and has been in good health. You decide to treat the patient with a prophylactic antibiotic. A. Amoxicillin/clavulanate (Augmentin) B. Azithromycin (Zithromax) C. Cephalexin (Keflex) D. Clindamycin (Cleocin) E. Metronidazole (Flagyl)

A. Amoxicillin/clavulanate (Augmentin) Prophylactic antibiotics should be given for all closed-fist injuries unless the skin has not been penetrated,and for puncture wounds caused by cat bites. The antibiotic should have both aerobic and anaerobicactivity and include Pasteurella coverage for animal bites and Eikenella coverage for human bites.Suggested regimens include amoxicillin/clavulanate. If the patient is allergic to penicillin, clindamycin pluslevofloxacin or moxifloxacin, which has anaerobic coverage, can be used. Azithromycin, cephalexin, andmetronidazole are not first-line antibiotics following a cat bite.

In addition to NSAIDs and narcotics for pain control, which one of the following would be most effective for hastening passage of the stone 6mm ?

A. An α1-blocker such as tamsulosin (Flomax)

An otherwise healthy 42-year-old male presents to your office with low back pain that started a week ago after he lifted a heavy box. Since the time of his injury he has been having consistent pain, numbness, and tingling that radiates down the back of his right leg to his calf. Which one of the following would you order at this time? (check one) A. No imaging B. Plain radiography C. CT D. MRI

A. No imaging Uncomplicated acute low back pain and/or radiculopathy is a benign, self-limited condition and earlyimaging is associated with worse overall outcomes and is likely to identify minor abnormalities even inasymptomatic patients. Imaging for acute low back pain should be reserved for cases that are suspiciousfor cauda equina syndrome, malignancy, fracture, or infection. In the absence of red flags such asprogressive motor or sensory loss, new urinary retention or overflow incontinence, a history of cancer,a recent invasive spinal procedure, or significant trauma relative to age, imaging is not warrantedregardless of whether radiculopathy is present, unless symptoms persist despite a trial of at least 6 weeks of medical management and physical therapy.

A 15-year-old male presents with a 2-day history of dark-colored urine, lower extremity edema, and fatigue. Approximately 2 weeks ago he said he had a "bad sore throat" that was treated empirically with amoxicillin. On examination his blood pressure is 144/92 mm Hg, his pulse rate is 76 beats/min, and his other vital signs are normal. Other than mild dependent edema there are no additional significant physical examination findings. A urinalysis dipstick shows 3+ hematuria.Which one of the following findings on microscopic evaluation of the urine sediment would help to confirm the diagnosis in this patient? (check one) A. Gram-positive cocci in chains B. RBC casts C. WBC casts D. Eosinophils E. Oxalate crystals

B. RBC casts This is a classic presentation for acute poststreptococcal glomerulonephritis (APSGN), with the onset ofgross hematuria associated with hypertension and systemic edema. This is most commonly seen inschool-age children, usually 1-2 weeks after an episode of pharyngitis or 3-4 weeks after an episode ofimpetigo, caused by so-called nephritogenic strains of Group A -hemolytic Streptococcus. The hematuriais caused by immune complex-mediated glomerular injury.Bacteriuria may be seen in both upper and lower urinary tract infections, but may also be a spuriousfinding, especially with the combined presence of epithelial cells. The classic finding on microscopicurinalysis for acute glomerulonephritis is the presence of RBC casts. WBC casts are seen with acutepyelonephritis. The presence of urinary eosinophils indicates acute interstitial nephritis. Calcium oxalatemakes up the most common type of kidney stones.Antibiotics prescribed for antecedent pharyngitis do not prevent APSGN. Treatment is supportive,controlling blood pressure and edema with a thiazide or a loop diuretic. The prognosis for resolution andfull recovery of the vast majority of patients with APSGN is excellent, especially in the pediatric age group.

A 54-year-old white male presents with drooping of his right eyelid for 3 weeks. On examination, he has ptosis of the right upper lid, miosis of the right pupil, and decreased sweating on the right side of his face. Extraocular muscle movements are intact. In addition to a complete history and physical examination, which one of the following would be most appropriate at this point? (check one) A. A chest radiograph B. MRI of the brain and orbits C. 131I thyroid scanning D. A fasting blood glucose level E. An acetylcholine receptor antibody level

A. A chest radiograph The clinical triad of Horner's syndrome-ipsilateral ptosis, miosis, and decreased facial sweating-suggests decreased sympathetic innervation due to involvement of the stellate ganglion, a complication of Pancoast's superior sulcus tumors of the lung. Radiographs or MRI of the pulmonary apices and paracervical area is indicated. Horner's syndrome may accompany intracranial pathology, such as the lateral medullary syndrome (Wallenbergs syndrome), but is associated with multiple other neurologic symptoms, so MRI of the brain is not indicated at this point. The acetylcholine receptor antibody level is a test for myasthenia gravis, which can also present with ptosis, but not with full-blown Horner's syndrome. Diabetes mellitus and thyroid disease do not commonly present with Horner's syndrome.

A 67-year-old male presents for a Medicare wellness visit. He underwent basic laboratory work prior to the office visit. He is feeling well and does not have any concerns or symptoms. His blood pressure is 127/76 mm Hg, his heart rate is 64 beats/min, and he is afebrile. A comprehensive metabolic panel is unremarkable. A CBC shows the following results:WBCs 7500/mm3 (N 4100-10,900)RBCs 4.05 million/mm3 (N 4.70-6.10)Hemoglobin 12.9 g/dL (N 14.0-18.0)Hematocrit 39% (N 42-52)Mean corpuscular volume 82 μm3 (N 80-95)Platelets 197,000/mm3 (N 130,000-448,000)Which one of the following would be the most appropriate next step in the workup of this patient? (check one) A. A serum ferritin level B. A serum transferrin receptor-ferritin index C. Oral iron supplementation, and a repeat CBC in 4 weeks D. Referral to a gastroenterologist

A. A serum ferritin level Anemia is often diagnosed incidentally on laboratory testing and is often asymptomatic. It is associatedwith increased morbidity and mortality in older adults, and is often caused by nutritional deficiencies,chronic kidney disease, occult blood loss from gastrointestinal malignancies, or chronic inflammation.However, in many patients the cause remains unknown. A detailed history and physical examination areindicated. In patients with normocytic or microcytic anemia, a serum ferritin level should be ordered. Alow serum ferritin level is associated with iron deficiency and should be further evaluated so the underlyingcause can be addressed. A serum transferrin-receptor-ferritin index should be determined for patients witha serum ferritin level between 46 and 100 ng/mL to distinguish between iron deficiency anemia and othertypes of anemia. Referring this patient to a gastroenterologist would not be indicated at this time.

A 65-year-old male with type 2 diabetes mellitus, hypertension, and obstructive sleep apnea sees you for follow-up. He does not use tobacco or other drugs, and his alcohol consumption consists of two drinks per day. His BMI is 31 kg/m2, and he just started a fitness program. The patient tells you that his brother was recently diagnosed with atrial fibrillation and he asks you if this increases his own risk.Which one of the following factors would increase the risk of atrial fibrillation in this patient? (check one) A. Alcohol use B. Treatment with lisinopril (Prinivil, Zestril) C. Treatment with pioglitazone (Actos) D. Use of a continuous positive airway pressure (CPAP) device E. Physical stress

A. Alcohol use

A 67-year-old male diagnosed with polymyalgia rheumatica is started on long-term prednisone therapy. Which one of the following is the recommended first-line agent to prevent steroid-induced osteoporosis? (check one) A. Alendronate (Fosamax) B. High-dose vitamin D C. Raloxifene (Evista) D. Teriparatide (Forteo)

A. Alendronate (Fosamax) Patients are at risk of developing glucocorticoid-induced osteoporosis if they are on long-term glucocorticoid therapy, defined as >2.5 mg of prednisone for a duration of 3 months or longer. The American College of Rheumatology recommends pharmacologic treatment for these patients, as well asfor patients receiving glucocorticoids who have a bone mineral density T-score -2.5 at either the spineor the femoral neck and are either male and 50 years of age or female and postmenopausal. Therapy isalso recommended in patients 40 years of age who do not meet these criteria but have a 10-year risk ofmajor osteoporotic fracture of at least 20% or a risk of hip fracture of at least 3% according to the FRAXtool.Oral bisphosphonates are recommended as first-line agents for preventing glucocorticoid-inducedosteoporotic fractures, although intravenous bisphosphonates can be used if patients are unable to use theoral forms. Supplementation of calcium (800-1000 mg) and vitamin D (400-800 IU) is also recommended.Raloxifene and teriparatide are options when bisphosphonate therapy fails or is contraindicated (SOR A). Ref: Buckley L, Humphrey MB: Glucocorticoid-induced osteoporosis. N Engl J Med 2018;379(26):2547-2556

While on call for your group practice you are called to admit a 23-year-old female with a history of sickle cell disease who presented to the emergency department with chest pain, a cough, and shortness of breath. She has no history of recent hospitalization.Physical Findings Blood pressure ..................................... 176/86 mm Hg Pulse.........................................................103 beats/min Respiratory rate ..................................... 20/min Temperature............................................ 37.8°C (100.0°F) Oxygen saturation................................... 89% on room air A chest radiograph shows consolidation in the right lower lobe.In addition to oxygen, intravenous fluids, an intravenous third-generation cephalosporin, and pain management, which one of the following is important to include in the patient's treatment plan? (check one) A. Azithromycin (Zithromax) B. Daptomycin (Cubicin) C. Ertapenem (Invanz) D. Gentamicin E. Vancomycin (Vancocin)

A. Azithromycin (Zithromax) This patient has acute chest syndrome (ACS), a serious vaso-occlusive complication of sickle cell disease (SCD). Its cause may be multifactorial, but infections are common and antimicrobials are indicated. However, the clinical course of ACS is significantly different from infectious pneumonia in patients without SCD, due to the damaged microvasculature that occurs in ACS. Studies have shown that atypical pathogens predominate in ACS and it is therefore important to treat all patients with ACS with antibiotics that cover Mycoplasma and Chlamydophila . Viral infections are also common, especially in children with ACS. Other possible pathogens include Staphylococcusaureus , Streptococcus pneumoniae , and Haemophilus influenzae . Therefore, the use of a third-generation cephalosporin along with azithromycin is the recommended antibiotic coverage.

An 8-year-old male is brought to your office because of a rash on his arms and legs that has been present for the last several weeks. It seems to be spreading gradually, according to his parents. No other family members have similar symptoms. A physical examination reveals excoriated erythematous papules on both forearms and both lower legs that drain a small amount of serous fluid. The child says that these are itchy. There are no signs of deeper inflammation and no lesions are present on the scalp, hands, thorax, or groin.Which one of the following is the most likely cause of this problem? (check one) A. Bedbugs B. Brown recluse spiders C. Mites/chiggers D. Scabies E. Ticks

A. Bedbugs This child's presentation appears most consistent with bites from an insect. Having multiple exposures on skin often not covered by clothing would be typical of household fleas or bedbugs. Tick bites are typically identified by the presence of an actively feeding insect or a single papular lesion. Similarly, brown recluse spider bites would not be expected to be multiple or recurrent. A chigger is the larval form of a mite, which is an eight-legged arthropod. The larval form has only six legs, and tends to crawl into spaces near constricted clothing and cause welts from their bites along the neckline, waistline, sock line, or more rarely on the genitals. A scabies infestation often presents as an eczematous rash in semiprotected folds of skin such as the web spaces of fingers, the umbilicus, the axillae, or the genital region.

You perform the initial newborn examination on a male on his first day of life, following an uncomplicated vaginal delivery at an estimated gestational age of 37 weeks and 6 days. The prenatal course was significant for the initial presentation for prenatal care at 22 weeks gestation. You note that the infant's upper lip is thin and the philtrum is somewhat flat.Which additional finding would increase your concern for fetal alcohol syndrome? (check one) A. Curvature of the fifth digit of the hand (clinodactyly) B. A supernumerary digit of the hand C. Flattening of the head (plagiocephaly) D. Metatarsus adductus in one foot E. Syndactyly of the toes (webbed feet)

A. Curvature of the fifth digit of the hand (clinodactyly) In addition to clinodactyly, fetal alcohol syndrome is associated with camptodactyly (flexion deformity of the fingers), other flexion contractures, radioulnar synostosis, scoliosis, and spinal malformations. It is also associated with many neurologic, behavioral, and cardiovascular abnormalities, as well as other types of abnormalities. Plagiocephaly, supernumerary digits, syndactyly, and metatarsus adductus are common in newborns but are not related to fetal alcohol spectrum disorders.

A 55-year-old patient with a history of alcoholism is admitted through the emergency department with acute pancreatitis. Which one of the following tests performed at the time of admission can best predict the severity of pancreatitis? A. Hematocrit B. C-reactive protein C. Serum amylase D. Serum lipase E. CT of the abdomen

A. Hematocrit Knowing the severity of pancreatitis helps predict how aggressive management should be. Hematocrit,BUN, and creatinine levels are the most useful predictors of the severity of pancreatitis, reflecting thedegree of intravascular volume depletion. C-reactive protein is often elevated, but it is not as useful ashematocrit for predicting severity. Serum amylase and lipase have no prognostic value. CT evidence ofsevere pancreatitis lags behind clinical and laboratory evidence, and early CT underestimates the severityof the acute process.

A 75-year-old male presents with a 12-month history of chest pressure radiating to his left arm that occurs predictably after he walks briskly for 2 blocks and goes away with rest. A treadmill stress test suggests coronary artery disease. The patient would prefer medical therapy over revascularization if possible.The patient's blood pressure is 120/85 mm Hg. His heart rate is 52 beats/min and has been in the low 50s at past visits. You initiate daily aspirin and a high-intensity statin, and prescribe sublingual nitroglycerin to use as needed for chest pain.Which one of the following additional treatments is recommended for management of his angina? (check one) A. Isosorbide mononitrate B. Ivabradine (Corlanor) C. Metoprolol succinate (Toprol-XL) D. Ranolazine (Ranexa) E. Verapamil (Calan)

A. Isosorbide mononitrate In addition to aspirin, a high-intensity statin, and sublingual nitroglycerin as needed, patients with chronic stable angina may be treated with β-blockers, calcium channel blockers, and/or long-acting nitrates. β-Blockers and heart rate-lowering calcium channel blockers should be avoided in this patient who already has bradycardia. Ranolazine, which affects myocardial metabolism, is not used as a first-line agent. Ivabradine is not a first-line agent and is used only in patients with heart failure. A long-acting nitrate or a dihydropyridine calcium channel blocker would be appropriate for this patient.

A 9-year-old female presents with a 4-week history of right knee pain with activity. There is no history of trauma or recent illness. Your examination reveals lateral patellar tracking with extension of the knee.Which one of the following is the most likely diagnosis? (check one) Which one of the following is the most likely diagnosis? (check one) A. Patellofemoral pain syndrome B. Osgood-Schlatter disease C. Growing pains D. Patellar tendinopathy E. Sever's disease

A. Patellofemoral pain syndrome Patellofemoral pain syndrome is one of the most common causes of knee pain in children, particularly adolescent girls. Pain beneath the patella is the most common symptom. Squatting, running, and other vigorous activities exacerbate the pain. Walking up and down stairs is a classic cause of the pain, and pain with sitting for an extended period is also common. The physical examination reveals isolated tenderness with palpation at the medial and lateral aspects of the knee, and the grind test is also positive.Osgood-Schlatter disease is seen in skeletally immature patients. Rapid growth of the femur can cause tight musculature in the quadriceps across the knee joint. It typically appears between the ages of 10 and 15, during periods of rapid growth. Pain and tenderness over the tibial tubercle and the distal patellar tendon is the most common presentation. The pain is aggravated by sports participation, but also occurs with normal daily activities and even at rest.Growing pains most often affect the thigh and quadriceps and occur during late afternoon or evening, or wake the patient at night. The joints are not affected. The pain typically goes away by morning, and may sometimes occur the day after vigorous or unusual activity.Patellar tendinopathy is an overuse injury often seen in those who participate in jumping sports such as volleyball, and is also related to frequent stops and starts in football players. It typically causes infrapatellar pain, and findings include extensor mechanism malalignment, weakness of ankle flexors, and tightness of the hamstring, heel cord, and/or quadriceps.Sever's disease is an overuse syndrome most often seen between the ages of 9 and 14, and is related to osteochondrosis at the insertion of the Achilles tendon on the calcaneal tuberosity. It occurs during periods of rapid growth, causes heel pain during and after activity, and is relieved with rest. It is often related to beginning a new sport or the start of a season.

A 32-year-old female presents with heat intolerance, excessive weight loss, and anxiety. She gave birth 6 months ago and recently stopped breastfeeding. On examination her thyroid gland is slightly diffusely enlarged and nontender. Laboratory studies reveal a decreased TSH level and elevated free T3 and T4 levels. You suspect that she has postpartum thyroiditis.Which one of the following tests would be most useful to confirm the diagnosis? (check one) A. Radioactive iodine uptake B. Thyroid peroxidase antibody levels C. Thyroid ultrasonography D. Thyrotropin receptor antibody levels

A. Radioactive iodine uptake

A 7-year-old female is brought to your office by her mother, who says that the child has developed underarm odor and is beginning to develop acne. The patient has an unremarkable history, was born at full term, and has no chronic medical problems. There is no family history of endocrine disorders or precocious puberty.An examination reveals normal vital signs and a normal BMI, and her height is stable on the growth curve with no increased velocity. Her sexual maturity rating is stage 1. She has some open and closed comedones on her forehead and back. Which one of the following would be most appropriate at this point? (check one) A. Reassurance and surveillance over the next 3-6 months B. TSH, LH, and FSH levels C. Bone age radiography D. Bone age radiography and TSH, LH, and FSH levels

A. Reassurance and surveillance over the next 3-6 months Premature adrenarche without development of secondary sex characteristics is usually idiopathic and doesnot lead to an abnormal pattern of development. Reassurance and surveillance over the next 3-6 monthswould be most appropriate at this time. Laboratory studies and radiography warrant consideration if thepatient develops secondary sex characteristics before the age of 8, or if her height velocity increasesrapidly during the surveillance period.

A 64-year-old male presents to your office after vomiting blood twice over the last 2 hours. He is healthy otherwise, except for hyperlipidemia. On the initial examination he has mild hypotension and tachycardia, which normalize after fluid resuscitation. He has no further vomiting episodes.What level of hemoglobin should be the threshold for transfusion of red cells in this situation? A. 6.0 g/dL B. 7.0 g/dL C. 8.0 g/dL D. 9.0 g/dL E. 10.0 g/dL

B. 7.0 g/dL In otherwise healthy stable patients with upper gastrointestinal bleeding, a transfusion of red cells is recommended when the hemoglobin level falls below 7.0 g/dL. In hypotensive patients with severe bleeding, a blood transfusion before the hemoglobin level reaches 7.0 g/dL is needed to prevent significant decreases below this level that would occur with just fluid resuscitation. In hemodynamically stable patients with known cardiovascular disease and significant upper gastrointestinal bleeding, 8.0 g/dL should be the threshold for a blood transfusion.

A 50-year-old male sees you for a health maintenance visit. He has a 20-year history of smoking 1 pack of cigarettes per day but his history is otherwise unremarkable. He is not aware of any disease that runs in his family, including diabetes mellitus. He has not visited a physician for the past 5 years.A physical examination reveals a BMI of 28.2 kg/m², normal blood pressure, and no other significant findings. Laboratory testing reveals a fasting plasma glucose level of 107 mg/dL.According to the U.S. Preventive Services Task Force, which one of the following would be most appropriate at this point? (check one) A. No further diabetes screening unless his BMI increases to ≥30 kg/m² B. A repeat fasting plasma glucose level in 1-2 weeks C. A repeat fasting plasma glucose level in 1 year D. A repeat fasting plasma glucose level in 3 years E. Treatment with metformin (Glucophage)

B. A repeat fasting plasma glucose level in 1-2 weeks The U.S. Preventive Services Task Force (USPSTF) recommends screening for abnormal blood glucose levels as part of a cardiovascular disease risk assessment for adults who are 40-70 years of age and who are overweight (BMI 25.0-29.9 kg/m²) or obese (BMI ≥30.0 kg/m²). Since his fasting blood glucose result was in the range consistent with impaired fasting glucose (100-125 mg/dL), the USPSTF recommends confirming the diagnosis of the abnormal glucose level soon by performing the same test on a different day. Appropriate treatment should begin once the diagnosis is confirmed.

An 11-year-old female is referred to you after a sports physical examination because 2+ protein was found on a random dipstick urinalysis. She feels well and does not have any health concerns. She plays soccer an average of 5 days a week.The patient's medical history is unremarkable and she takes no medications. Menarche has not occurred. She does not report any urinary or back symptoms, recent illness, edema, or weight change. A physical examination is normal. A dipstick urinalysis in your office shows 1+ protein but is otherwise normal.Which one of the following would you recommend next? (check one) A. Withdrawing from all physical activity for 24 hours and a 24-hour urine for protein B. A spot protein/creatinine ratio performed on first morning urine C. Serum BUN, creatinine, electrolyte, and albumin levels D. Ultrasonography of the kidneys and bladder E. Referral to a pediatric nephrologist

B. A spot protein/creatinine ratio performed on first morning urine It is important to distinguish serious illness from benign causes of proteinuria, which are the most common etiology in children. Confirming the presence of proteinuria is the next step in this case because functional (exercise/stress-induced) and orthostatic proteinuria are common types of proteinuria and are transient. A 24-hour urine for protein is a possible option, but would be impractical and burdensome for a healthy-acting 11-year-old. The pediatric nephrology panel of the National Kidney Foundation reported that a spot protein/creatinine ratio is a reliable test for ruling out proteinuria. A specialist referral, blood analysis, and ultrasonography are unnecessary unless persistent proteinuria is identified.

A 53-year-old male sees you for follow-up of his hypertension. His medical history includes prediabetes and gout, and he is currently taking lisinopril (Prinivil, Zestril), 40 mg daily, to control his blood pressure. His blood pressure after resting is 148/86 mm Hg. Laboratory findings include a serum creatinine level of 0.8 mg/dL (N 0.6-1.2) and a serum potassiumlevel of 4.5 mEq/L (N 3.5-5.1). Which one of the following would be the most appropriate management of this patient's hypertension? (check one) A. No change in medication B. Add amlodipine (Norvasc) C. Add hydrochlorothiazide D. Add losartan (Cozaar) E. Add metoprolol succinate (Toprol-XL)

B. Add amlodipine (Norvasc) First-line agents for hypertension include ACE inhibitors, angiotensin receptor blockers, thiazide diuretics, and calcium channel blockers. Hydrochlorothiazide would be relatively contraindicated due to the patient's gout. Losartan, an angiotensin receptor blocker, should not be added because the patient is taking an ACE inhibitor. Metoprolol succinate, a β-blocker, is not a first-line agent for blood pressure unless there is another indication such as systolic heart failure or migraine prophylaxis.

A 42-year-old male with a history of alcohol abuse was admitted to the hospital last night with mid-epigastric pain and tenderness. The hospital evaluation included an elevated lipase level and a normal ultrasound examination. He was diagnosed with acute pancreatitis, placed on NPO status, and started on intravenous fluids and pain control. This morning he still has moderate epigastric pain and tenderness, and mild nausea but no vomiting. He says he would like to try eating some food. Which one of the following would you recommend at this time? (check one) A. Remaining NPO until the pain and tenderness have improved B. Allowing the patient an oral diet as tolerated C. Initiating nasogastric feeding D. Initiating nasojejunal tube feeding E. Initiating total parenteral nutrition

B. Allowing the patient an oral diet as tolerated The American Gastroenterological Association guidelines on acute pancreatitis recommend initiating oralfeedings early in the course in order to protect the gut-mucosal barrier, which may limit infectiouscomplications and does not increase hospital length of stay or other complications. Nasogastric ornasojejunal tube feeding may be considered at 3-5 days if oral feedings are not tolerated. Total parenteralnutrition is indicated only when enteral feedings cannot supply adequate caloric intake or are not possiblefor other reasons. The incidence of single or multiple organ failure or infected necrosis is significantlyincreased with the use of total parenteral nutrition.

A 26-year-old gravida 2 para 1 presents at 30 weeks gestation with a complaint of severe itching. She has excoriations from scratching in various areas. She says that she had the same problem during her last pregnancy, and her medical records reveal a diagnosis of intrahepatic cholestasis of pregnancy. Elevation of which one of the following is most characteristic of this disorder? (check one) A. γ-Glutamyltransferase (GGT) B. Bile acids C. Direct bilirubin D. Indirect bilirubin E. Prothrombin time

B. Bile acids Intrahepatic cholestasis of pregnancy classically presents as severe pruritus in the third trimester. Characteristic findings include the absence of primary skin lesions and elevation of serum levels of total bile acids. Jaundice and elevated bilirubin levels may or may not be present. The GGT usually is normal or modestly elevated, which can help differentiate this condition from other cholestatic liver diseases. The prothrombin time usually is normal, but if elevated it may reflect a vitamin K deficiency from malabsorption

A healthy 49-year-old female presents to your office for a routine health maintenance visit. Since her last visit a year ago she has had only two menstrual periods. She reports sudden sensations of extreme heat in her face, neck, and chest that last just a few minutes but occur throughout the day. These symptoms are very bothersome and interfere with the quality of her sleep. Which one of the following would you recommend to relieve her symptoms? (check one) A. Black cohosh B. Combined estrogen and progesterone C. Compounded bioidentical hormones D. Micronized progesterone E. Testosterone

B. Combined estrogen and progesterone This patient presents with typical vasomotor symptoms that can begin in perimenopause and affect sleepquality. Hormone therapy is the gold standard for treatment of vasomotor symptoms. Combination estrogenand progesterone therapy is highly effective for vasomotor symptoms and provides protection againstuterine neoplasia

A 62-year-old female who is a new patient requests a thyroid evaluation because she has a history of abnormal thyroid test results. You obtain a copy of her records, which include a TSH level of 0.2 μU/mL (N 0.4-4.2) and a free T4 level of 2.0 ng/dL (N 0.8-2.7) from 3 years ago. She reports feeling well and has no other health conditions. She does not take any medications.A physical examination reveals normal vital signs, a BMI of 23 kg/m2, no neck masses, a normal thyroid size, and normal heart sounds. Laboratory studies reveal a TSH level of 0.1 μU/mL, a free T4 level of 2.5 ng/dL, and a free T3 level of 3.1 pg/mL (N 2.3-4.2).Treatment for this condition would be indicated if the patient has an abnormal (check one) Treatment for this condition would be indicated if the patient has an abnormal (check one) A. calcium level B. DXA scan C. glucose level D. lipid level E. thyroid ultrasonography study

B. DXA scan This patient has subclinical hyperthyroidism as evidenced by her low TSH level with normal free T4 and free T3 levels. Common causes of subclinical hyperthyroidism include Graves disease, autonomous functioning thyroid adenoma, and multinodular toxic goiter. Subclinical hyperthyroidism may progress to overt hyperthyroidism; this is more likely in patients with TSH levels <0.1 μU/mL. Even in the absence of overt hyperthyroidism these patients are at higher risk for several health conditions, including atrial fibrillation, heart failure, and osteoporosis. For this reason it is important to assess for these conditions and consider treating the underlying thyroid condition, as well as the complication. The American Thyroid Association recommends treating patients with complications who are either over age 65 or have a TSH level <0.1 μU/mL.

A 70-year-old male presents to your office for follow-up after he was hospitalized for acute coronary syndrome. He has not experienced any pain since discharge and is currently in a supervised cardiac rehabilitation exercise program. His medications include aspirin, lisinopril (Prinivil, Zestril), and metoprolol, but he was unable to tolerate atorvastatin (Lipitor), 40 mg daily, because he developed muscle aches. Which one of the following would you recommend? (check one) A. Evolocumab (Repatha) B. Ezetimibe/simvastatin (Vytorin) C. Fenofibrate (Tricor) D. Niacin E. Omega-3 fatty acid supplements

B. Ezetimibe/simvastatin (Vytorin) High-intensity statin therapy is recommended for patients younger than 75 years of age with knowncoronary artery disease. For those who are intolerant of high-intensity statins, a trial of amoderate-intensity statin is appropriate. There is evidence to support ezetimibe plus a statin in patients withacute coronary syndrome or chronic kidney disease. Omega-3 fatty acids, fibrates, and niacin should notbe prescribed for primary or secondary prevention of atherosclerotic cardiovascular disease because theydo not affect patient-oriented outcomes. PCSK9 inhibitors such as evolocumab are injectable monoclonalantibodies that lower LDL-cholesterol levels significantly and have produced some promising results, butmore studies are needed to determine when this would be cost effective.

A 24-year-old male complains of feeling on edge all of the time. For the past 2 years he has had difficulty controlling his worrying about work, school, and relationships. He has had more difficulty concentrating at work and school, is more irritable, and has difficulty staying asleep all night. He drinks alcohol moderately and does not use drugs. You recommend regular exercise and refer him to a therapist for cognitive-behavioral therapy to help manage his symptoms.Which one of the following would be first-line medical therapy for this patient? (check one) A. Bupropion (Wellbutrin) B. Fluoxetine (Prozac) C. Lorazepam (Ativan) D. Methylphenidate (Ritalin, Concerta) E. Quetiapine (Seroquel)

B. Fluoxetine (Prozac) Though symptoms of generalized anxiety disorder (GAD) overlap with other psychiatric and medical conditions, the case presented is most consistent with GAD. SSRIs are first-line therapy for GAD (SOR B). Benzodiazepines such as lorazepam can improve anxiety-related symptoms, but due to the side effects and addiction potential they are recommended for short-term use (SOR B). Bupropion is approved for the treatment of depression but is not used to treat GAD. Quetiapine may be considered as second-line therapy for GAD (SOR B). Methylphenidate is first-line therapy for attention-deficit/hyperactivity disorder but is not indicated to treat GAD. Psychotherapy, especially cognitive-behavioral therapy, is also first-line treatment for GAD (SOR A), and exercise can also improve symptoms (SOR B).Ref: Locke AB, Kirst N, Shultz CG: Diagnosis and management of generalized anxiety disorder and panic disorder in adults. Am Fam Physician 2015;91(9):617-624.

A 14-year-old male sees you for a well child examination. He had one dose of HPV vaccine at his last well child examination 1 year ago.Which one of the following is true regarding HPV vaccine for this patient? (check one) A. He does not require additional HPV vaccine B. He should receive one dose of the vaccine now and no additional HPV vaccine in the future C. He should receive the vaccine now and again in 4 months D. He should receive the vaccine now and again in 6 months E. He should receive the vaccine now, in 2 months, and in 4 months

B. He should receive one dose of the vaccine now and no additional HPV vaccine in the future HPV vaccine is currently recommended for males and females at age 11. Catch-up vaccination isrecommended until age 21 in males and 26 in females. Children who receive the first dose of the vaccinebefore the age of 15 and receive two doses are considered adequately vaccinated. If the first dose is givenafter age 15, a three-dose series is recommended.

A 59-year-old male reports nausea, vomiting, and progressive fatigue for the past few months. At his last visit, 6 months ago, his blood pressure was poorly controlled and hydrochlorothiazide was added to his β-blocker therapy. At this visit he appears moderately dehydrated on examination. Laboratory testing reveals a serum calcium level of 12.5 mg/dL (N 8.0-10.0), a BUN level of 36 mg/dL (N 6-20), and a creatinine level of 2.2 mg/dL (N 0.6-1.1). A CBC, albumin level, and electrolyte levels are normal. His intact parathyroid hormone level is reported a few days later, and is 60 pg/mL (N 10-65). What is the most likely cause of his hypercalcemia? (check one) A. Renal failure B. Hyperparathyroidism C. Milk alkali syndrome D. Sarcoidosis

B. Hyperparathyroidism Many patients have mild hyperparathyroidism that becomes evident only with an added calcium load. Thiazide diuretics reduce calcium excretion and can cause overt symptoms in a patient whose hyperparathyroidism would otherwise have remained asymptomatic. The finding of a normal parathyroid hormone (PTH) level in a patient with hypercalcemia is diagnostic for hyperparathyroidism, since PTH should be suppressed in the presence of elevated calcium. Symptomatic hypercalcemia causes dehydration because of both intestinal symptoms and diuresis. Reversible renal insufficiency can result, and can become permanent if it is long-standing and severe. Conversely, renal failure usually causes hypocalcemia, but can cause hypercalcemia resulting from tertiary hyperparathyroidism. This develops after severe hyperphosphatemia and vitamin D deficiency eventually produce hypersecretion of PTH. This patient's renal insufficiency is not severe enough to cause tertiary hyperparathyroidism. Milk alkali syndrome is hypercalcemia resulting from a chronic overdose of calcium carbonate, and is becoming more common as more patients take calcium and vitamin D supplements. In milk alkali syndrome, and other causes of hypercalcemia such as sarcoidosis, the PTH level is appropriately suppressed. Ref: Carroll MF, Schade DS: A practical approach to hypercalcemia.

A 48-year-old female presents to the emergency department with chest pain. The evaluation, including CT angiography, reveals a pulmonary embolus.Which one of the following initial findings would be the strongest indication for thrombolytic therapy? (check one) A. Elevated troponin B. Hypotension C. Hypoxia D. Bilateral pulmonary emboli E. Right ventricular dysfunction on echocardiography

B. Hypotension Recent guidelines have suggested that hypotension (a systolic blood pressure <90 mm Hg or a diastolic blood pressure <60 mm Hg, for 15 minutes or longer) should be treated with thrombolysis in patients who are not at high risk for bleeding. Patients who have other indicators of cardiopulmonary impairment without signs of hypotension should be given anticoagulation therapy and aggressive supportive care, but should not be treated with thrombolytic therapy. If the patient's condition continues to deteriorate as evidenced by the development of hypotension or other clinical indicators of cardiopulmonary compromise, thrombolysis may be considered.

A gravida 2 para 0 at 34 weeks gestation presents to your office because of diffuse itching. She does not have any known allergies other than seasonal allergies, and she does not have any new contacts. An examination is normal other than some scattered excoriations, and there is no other distinct rash. She has tried moisturizers but her symptoms have not improved. Which one of the following would be most appropriate at this point? (check one) A. Monitoring for the development of a rash B. Liver function tests and serum bile acid levels C. Topical corticosteroids D. Oral antihistamines E. Varicella-zoster immune globulin

B. Liver function tests and serum bile acid levels Whenever a pregnant woman presents with pruritus without a primary rash, it is important to evaluate herfor intrahepatic cholestasis of pregnancy. This diagnosis is associated with increased fetal mortality andwarrants increased antenatal surveillance as well as possible induction by 35-37 weeks gestation. It is mostappropriate to check for elevation of liver function tests and serum bile acids. Emollients, topicalcorticosteroids, and oral antihistamines can all be helpful for pruritus and certain rashes, but in this patientit is most important to promptly look for the cause of the pruritus. Varicella-zoster immune globulin wouldbe indicated if she had no immunity to varicella and had been exposed to varicella or if she had a rash thatwas suspected to be chickenpox.

Which one of the following is associated with testosterone supplementation in men with hypogonadism? (check one) A. Muscle wasting B. Polycythemia C. Osteoporosis D. An increased risk of benign prostatic hypertrophy

B. Polycythemia Testosterone increases hematocrit and can cause polycythemia. In patients receiving testosterone supplementation, hematocrit should be monitored every 6 months for the first 18 months, then annually. Testosterone should be discontinued if there is more than a 50% rise in hematocrit. Testosterone also causes an increase in lean body mass, and may increase bone density. Ref: Margo K, Winn R: Testosterone treatments: Why, when, and how?

A 36-year-old female calls your office because of a 2-day history of dysuria, urinary urgency, and urinary frequency. She has not had any fever, nausea, or vaginal discharge. She tells you her symptoms are similar to a previous urinary tract infection. She uses subdermal etonogestrel (Nexplanon) for contraception, takes no oral medications, and has no drug allergies.Which one of the following would be most appropriate at this point? (check one) A. Prescribe ciprofloxacin (Cipro) B. Prescribe nitrofurantoin (Macrobid, Macrodantin) C. Prescribe a urinary analgesic such as phenazopyridine (Pyridium) D. Ask the patient to come in today for evaluation E. Ask the patient to submit a urine specimen before you prescribe antibiotics

B. Prescribe nitrofurantoin (Macrobid, Macrodantin) There are three first-line antibiotics for uncomplicated UTI. These include nitrofurantoin for 5 days, trimethoprim/sulfamethoxazole for 3 days, and fosfomycin as a single dose (SOR A). Fluoroquinolones are second-line agents and are best reserved for more serious infections such as pyelonephritis.

You see a 12-year-old female for a well child check. She is healthy without any medical problems and neither she nor her father have any concerns today. She is up to date on her immunizations except for her third dose of HPV vaccine. She received the first dose of the vaccine at her 11-year-old well child check and her second dose 1 month later. However, she was told by a few friends that they only needed to get two doses of HPV vaccine, so she is very excited that she does not need any shots today. According to the CDC, which one of the following is true regarding HPV vaccine for this patient? (check one) A. She has completed her HPV vaccine series B. She should receive a third dose of HPV vaccine today C. She needs an HPV booster at age 21 but does not need a third dose of vaccine today D. She should have HPV titers drawn today and receive a third dose of vaccine only if the titers are low

B. She should receive a third dose of HPV vaccine today In 2016 the CDC changed the recommendation for the number of HPV vaccine doses for children ages 11-14. Children in this age group need only two doses of HPV vaccine 6-12 months apart. However, if they received two doses of HPV vaccine less than 5 months apart, they still need to have the third dose. Children and young adults over the age of 14 and those with certain immunocompromising conditions still require three doses of HPV vaccine. There is no indication for a booster dose at a later date, nor is there clinical data to support using titers to gauge immunogenicity to HPV.

A 42-year-old female presents with a 2-month history of right-sided shoulder pain. A historyreveals that her job requires repetitive motion, including abduction of the shoulder. Ibuprofenhas not been helpful and the pain interferes with her sleep. The physical examination suggests rotator cuff tendinitis. A radiograph of the shoulder is normal.You discuss treatment options and the patient decides to proceed with a corticosteroid injection.Which one of the following is the appropriate anatomic location for the injection? (check one) A. The acromioclavicular joint B. The subacromial space C. The intra-articular shoulder joint under fluoroscopy D. The area of insertion of the deltoid muscle E. The area of insertion of the long head of the biceps

B. The subacromial space

GLP-1 agonists such as exenatide (Byetta) can be used as second-line agents to help improve glycemic control in patients with type 2 diabetes mellitus. Which one of the following is a CONTRAINDICATION to their use? A. Hypothyroidism B. Thyroid cancer C. Coronary artery disease D. Heart failure

B. Thyroid cancer

You see a 53-year-old female with diabetes mellitus, hypertension, mixed hyperlipidemia, and GERD. Recent laboratory studies include an incidental finding of thrombocytopenia. The patient has no other significant past medical history, and she does not use tobacco or drink alcohol. Her current medications include metformin (Glucophage), lisinopril (Prinivil, Zestril), omeprazole (Prilosec), calcium citrate, and pravastatin (Pravachol). A physical examination is notable for a BMI of 31 kg/m2. Her skin, heart, lungs, abdomen, and extremities are normal. Results of a CBC and a comprehensive metabolic panel are normal with the following exceptions:Platelets 70,000 (N 150,000-379,000)Glucose 108 mg/dLBilirubin 0.4 mg/dL (N 0.0-0.4)Alkaline phosphatase 175 U/L (N 38-126)ALT (SGPT) 52 U/L (N 10-28)A peripheral smear is normal except for reduced platelets. Tests for hepatitis B, hepatitis C, and HIV are negative.The most likely etiology of this patient's thrombocytopenia is (check one) A. a hematologic malignancy B. chronic liver disease C. drug-induced thrombocytopenia D. immune thrombocytopenic purpura (ITP) E. primary bone marrow failure

B. chronic liver disease This patient presents with a typical example of nonalcoholic steatohepatitis (NASH) progressing toward cirrhosis, with multiple risk factors including diabetes mellitus, hyperlipidemia, obesity, and mildly elevated hepatic transaminases. Abnormalities of other cell lines would likely occur if a hematologic malignancy or bone marrow failure were present. While immune thrombocytopenic purpura is a diagnostic consideration, it is much less common than NASH and requires other causes to be ruled out. This patient is not taking any medications that have been frequently reported to cause drug-induced thrombocytopenia.

A 69-year-old female presents with scaling, redness, and irritation under her breasts for the past several months. She has tried several over-the-counter antifungal creams without any improvement. On examination you note erythematous, well demarcated patches with some scale under both breasts. You examine the rash with a Wood's lamp to confirm your suspected diagnosis.This rash is most likely to fluoresce A. bright yellow B. coral pink C. lime green D. pale blue E. totally white

B. coral pink A Wood's lamp may assist with the diagnosis of certain skin conditions. This patient's presentation is consistent with erythrasma caused by a Corynebacterium minutissimum infection, and use of an ultraviolet light would reveal a coral pink color. Pale blue fluorescence occurs with Pseudomonas infections, yellow with tinea infections, and totally white with vitiligo. A lime green fluorescence is not characteristic of a particular skin condition.

A 1-day-old newborn is brought to your office for a routine examination. His parents report that he is well. The prenatal course and delivery were unremarkable. An examination is normal except for a 1-cm wide dimple on the sacrum, 1 cm superior to the anus. The dimple has a tuft of dark hair. At this point you would recommend (check one) A. a follow-up examination in 1 month B. ultrasonography C. MRI D. a fistulogram/sinogram E. a dermatology consultation

B. ultrasonography Recognizing clinically significant abnormalities on the newborn examination is important. Newborns withsmall sacral dimples located far from the anal verge, without other skin findings such as hair, do not needimaging to rule out spinal dysraphism (tethered cord). While the exact parameters of what is consideredlarge (>0.5 cm diameter) and close (within 2.5 cm of the anal verge) can easily be found in referencematerials, the dimple described here is clearly concerning and needs imaging. Ultrasonography canaccurately and safely detect spinal dysraphism in these cases

A 32-year-old obese female from Saudi Arabia presents with muscle aches. Her clothing limits sun exposure and you decide to test her for vitamin D deficiency Which one of the following is the most appropriate test for this condition? (check one) A. Calcium B. Alkaline phosphatase C. 25-hydroxyvitamin D D. 1,25-dihydroxyvitamin D E. Parathyroid hormone

C. 25-hydroxyvitamin D Recommendations to screen for vitamin D deficiency apply only to patients at risk and not to the general population. This patient's obesity and her clothing style, which limits sun exposure to the skin, puts her at increased risk. Additionally, this patient's muscle aches may be a symptom of vitamin D deficiency. The recommended test for this condition is a 25-hydroxyvitamin D level. A 1,25-dihydroxyvitamin D level is recommended to monitor, not diagnose, certain conditions. Parathyroid hormone, calcium, and alkaline phosphatase levels are poor indicators of vitamin D status.

A healthy 35-year-old female presents to your office to discuss an upcoming trip to Bangladesh. She currently feels well and has no health problems. She is a nurse and will be traveling with a church group to work in a clinic for 1 month. This area is known to have a high prevalence of tuberculosis (TB). She is worried about contracting TB while she is there and asks for recommendations regarding TB screening. She had a negative TB skin test about 1 year ago at work. A TB skin test today is negative. Assuming she remains asymptomatic, which one of the following would you recommend? (check one) A. Prophylactic treatment with isoniazid starting 1 month prior to departure and continuing throughout her trip B. Prophylactic treatment with rifampin (Rifadin) starting 1 month prior to departure and continuing throughout her trip C. A repeat TB skin test 2 months after she returns D. A chest radiograph 2 months after she returns E. An interferon-gamma release assay (IGRA) 6 months after she returns

C. A repeat TB skin test 2 months after she returns Individuals who travel internationally to areas with a high prevalence of tuberculosis (TB) are at risk forcontracting the disease if they have prolonged exposure to individuals with TB, such as working in a healthcare setting. The CDC recommends either a TB skin test or an interferon-gamma release assay prior toleaving the United States. If the test is negative, the individual should repeat the testing 8-10 weeks afterreturning. A chest radiograph in asymptomatic individuals or prophylactic treatment at any point is notrecommended. Isoniazid and rifampin are options for treatment of latent TB.

A nulliparous 34-year-old female comes to your office for evaluation of fatigue, hair loss, and anterior neck pain. These symptoms have been gradually worsening for the past few months. Her past medical history is unremarkable. She has gained 5 kg (11 lb) since her last office visit 18 months ago. Examination of the thyroid gland reveals tenderness but no discrete nodules. Her TSH level is 7.5 μU/mL (N 0.4-4.2), her T4 level is low, and her thyroid peroxidase antibodies are elevated.Which one of the following would be the most appropriate next step? (check one) A. Continue monitoring TSH every 6 months B. Begin thyroid hormone replacement and repeat the TSH level in 6-8 weeks C. Begin thyroid hormone replacement and repeat the TSH level along with a T3 level in 6-8 weeks D. Order ultrasonography of the thyroid E. Order fine-needle aspiration of the thyroid

C. Begin thyroid hormone replacement and repeat the TSH level along with a T3 level in 6-8 weeks This patient has thyroiditis with biochemical evidence for autoimmune (Hashimoto's) thyroiditis. The mostappropriate plan of care is to begin thyroid hormone replacement and monitor with a repeat TSH level 6-8weeks later. It is not necessary to include a T3 level when assessing the levothyroxine dose. There is noneed to routinely order thyroid ultrasonography when there are no palpable nodules on a thyroidexamination. Fine-needle aspiration may be necessary to rule out infectious thyroiditis when a patientpresents with severe thyroid pain and systemic symptoms.

A 64-year-old male presents with increasing dyspnea on exertion. He feels well otherwise and has no chronic medical problems. A physical examination is normal. Pulmonary function testing reveals normal spirometry, with no evidence of an obstructive or restrictive pattern. However, his lung carbon monoxide diffusing capacity (DLCO) is low. Based on these results, which one of the following is the most likely diagnosis? (check one) A. Asthma B. Bronchiectasis C. Chronic pulmonary emboli D. COPD E. Pulmonary fibrosis

C. Chronic pulmonary emboli Low diffusing capacity of the lungs for carbon monoxide (DLCO) with normal spirometry indicates a disease process that disrupts gas transfer in the lungs without causing lung restriction or airflow obstruction. Common causes include chronic pulmonary emboli, heart failure, connective tissue disease with pulmonary involvement, and primary pulmonary hypertension. Asthma, bronchiectasis, COPD, and pulmonary fibrosis are associated with abnormalities on spirometry.

A 69-year-old female with hypertension, hyperlipidemia, and coronary artery disease had a myocardial infarction 1 year ago that was treated with percutaneous stenting. She was recently diagnosed with atrial fibrillation and takes diltiazem (Cardizem) for rate control. She is also taking lisinopril (Prinivil, Zestril), atorvastatin (Lipitor), atenolol (Tenormin), and aspirin, 81 mg.Of the following, which option would be best for thromboembolism prevention in this patient? (check one) A. Continue the current medication regimen B. Continue the current medication regimen C. Discontinue aspirin and begin apixaban (Eliquis) D. Add clopidogrel (Plavix)

C. Discontinue aspirin and begin apixaban (Eliquis) This patient has a CHA2DS2-VASc score of 3 (hypertension, age 65-74, female), which classifies her ashigh risk for thromboembolism. Oral anticoagulation is indicated for patients with a score of 2 or more,who are at high risk for pulmonary embolism/deep vein thrombosis (PE/DVT) (SOR C). Patients with ascore of 0-1 have a low to medium risk and may use aspirin with or without clopidogrel.In patients with atrial fibrillation and stable coronary artery disease, novel oral anticoagulants are preferred(SOR A). They reduce the risk of reinfarction, stroke, and overall mortality in patients with a past historyof myocardial infarction, and also help prevent PE/DVT.

Which one of the following is the most reliable measure to protect children from lead toxicity in the United States? (check one) A. Anticipatory guidance for parents and caregivers during well child visits B. Checking the serum lead level after a known exposure C. Eliminating the sources of lead in the community D. Iron and calcium supplementation to reduce lead absorption E. Providing appropriate cleaning equipment to families with known lead in the home

C. Eliminating the sources of lead in the community

A healthy 43-year-old executive presents with problems falling asleep and staying asleep. Doxepin (Silenor) and extended-release melatonin have not helped.In addition to behavioral interventions, which one of the following would be the most appropriate pharmacologic therapy for this patient's insomnia at this time? (check one) A. Diphenhydramine (Benadryl) B. Doxylamine (Unisom) C. Eszopiclone (Lunesta) D. Olanzapine (Zyprexa) E. Zaleplon (Sonata)

C. Eszopiclone (Lunesta) Although behavioral interventions are the mainstay of treatment for insomnia, they often need to be supplemented by pharmacologic therapy. When both doxepin and extended-release melatonin fail to provide benefit, a member of the Z-drug class should be tried next. Among the Z-drugs only eszopiclone provides an early peak onset and a long half-life, with a 1-hour approximate time to peak and a 6-hour half-life. While zaleplon has an equally short time to peak of 1 hour, it also has a 1 hour half-life. Antihistamines, including diphenhydramine and doxylamine, as well as atypical antipsychotics such as olanzapine, are not indicated unless used primarily to treat another condition.

A 33-year-old female comes to your office for follow-up of irritable bowel syndrome. You ruled out other causes of her abdominal bloating, abdominal pain, and diarrhea at earlier visits. She has no change in symptoms, such as constipation or blood in her stool. She has resisted treatment in the past, but her symptoms are becoming more frequent and she would now like to consider treatment.Evidence shows that which one of the following would most likely be beneficial for this patient? (check one) Evidence shows that which one of the following would most likely be beneficial for this patient? (check one) A. Acupuncture B. Increased insoluble fiber in her diet C. Fluoxetine (Prozac), 20 mg daily D. Neomycin, 1000 mg every 6 hours for 7 days E. Polyethylene glycol (MiraLAX), 17 g daily

C. Fluoxetine (Prozac), 20 mg daily This patient has diarrhea-predominant irritable bowel syndrome (IBS). There are many treatments available, with varying degrees of evidence. SSRIs, along with tricyclic antidepressants, have been shown to decrease abdominal pain and improve global assessment scores in those with IBS. Polyethylene glycol is a treatment for constipation and would not help this patient. Acupuncture has not been shown to be superior to sham acupuncture in improving IBS symptoms. Neomycin has been shown to improve symptoms in constipation-predominant IBS but would not be helpful in diarrhea-predominant IBS. Soluble fiber such as psyllium improves symptoms and decreases abdominal pain scores in patients with IBS. Insoluble fiber has not been shown to improve any IBS outcomes.

An asymptomatic 40-year-old male presents for a routine examination and is found to have a total bilirubin level of 1.8 mg/dL (N ≤1.0) and an indirect bilirubin level of 1.3 mg/dL. He drinks 3-6 beers/week. An examination and laboratory tests, including a CBC and serum liver enzymes, are within normal limits. Which one of the following is true regarding the diagnosis? (check one) A. The most likely diagnosis is alcoholic liver disease B. The most likely diagnosis is Dubin-Johnson syndrome C. The most likely diagnosis is Gilbert syndrome D. Ultrasonography of the liver and gallbladder are necessary to make a diagnosis

C. Gilbert syndrome is an autosomal dominant disease characterized by indirect hyperbilirubinemia caused by impaired glucuronyl transferase activity. The workup includes studies to exclude hemolysis (CBC, reticulocyte count, and haptoglobin) and liver disease (AST, ALT, alkaline phosphatase, and prothrombin time). Alcoholic liver disease is associated with a greater elevation of AST than of ALT. Dubin-Johhnson syndrome is a benign liver disease distinguished by direct or conjugated hyperbilirubinemia. Imaging studies are not required to confirm Gilbert syndrome; such studies are more useful for conditions involving conjugated hyperbilirubinemia. Other causes of indirect hyperbilirubinemia include hematoma, infection, cardiac disease, rhabdomyolysis, living at high altitude, thyrotoxicosis, and some medications.

A 52-year-old male sees you for the first time for a health maintenance visit. He previously tested positive on a hepatitis C antibody test performed by another physician. He seeks your opinion regarding follow-up testing.Which one of the following tests would you suggest at this point? (check one) A. α-Fetoprotein B. Hepatitis B antigen C. Hepatitis C RNA D. Ultrasonography of the liver E. A liver biopsy

C. Hepatitis C RNA Screening for hepatitis C virus (HCV) with an anti-HCV antibody test is recommended for all adults at high risk of infection, and one-time screening is recommended in adults born between 1945 and 1965. If the anti-HCV antibody test result is positive, current infection should be confirmed with a qualitative HCV RNA test.

A 62-year-old female comes to your office for evaluation of pain in her right thumb andwrist associated with sewing. She does not have any injury, numbness, tingling, orweakness. An examination reveals an otherwise healthy-appearing female with normal vitalsigns and no deformity or swelling in her wrists or hands. She has tenderness to palpationat the first dorsal compartment over the radial styloid and has pain with active and passivestretching of the thumb tendons over the radial styloid. She is very worried that she willhave to stop sewing and asks if there is anything she could try to alleviate her symptoms.Which one of the following would be most appropriate at this point? A. Reassurance that it will likely improve on its own within about a year B. A corticosteroid injection into the first extensor compartment C. Immobilization in a thumb spica splint and an NSAID for 1-4 weeks D. Radiographs of the thumb and wrist E. Referral to an orthopedic surgeon

C. Immobilization in a thumb spica splint and an NSAID for 1-4 weeks De Quervain's tenosynovitis usually occurs with repeated use of the thumb and ischaracterized by pain in the radial wrist. The course is typically self-limited but can last forup to a year, so waiting would not be a good option for this patient who wants to continueher usual activities as soon as possible. Conservative therapy with immobilization andNSAIDs is recommended if there are no contraindications to NSAIDs. A corticosteroidinjection is helpful but is typically reserved for severe cases or if conservative therapy fails.Surgery may be beneficial but is generally not recommended unless the course is severe,given the natural history of resolution.

A 69-year-old male with type 2 diabetes mellitus, obesity, and a history of coronary artery disease sees you for follow-up of his diabetes. His hemoglobin A1c has increased to 8.7% despite therapy with metformin (Glucophage), 1000 mg twice daily, and insulin glargine (Lantus).Which one of the following additional medications would be most effective for reducing his blood glucose level and lowering his risk of cardiovascular events? (check one) A. Exenatide (Byetta) B. Glipizide (Glucotrol) C. Liraglutide (Victoza) D. Rosiglitazone (Avandia) E. Sitagliptin (Januvia)

C. Liraglutide (Victoza) Liraglutide, exenatide, and dulaglutide are all GLP-1 receptor agonists. Of these, only liraglutide has been shown to lower the risk of recurrent cardiovascular events and has received FDA approval for this indication. Glipizide (a sulfonylurea), rosiglitazone, and sitagliptin have not been associated with improved cardiovascular outcomes. Empagliflozin, an SGLT2 inhibitor, has also been associated with secondary prevention of cardiovascular disease.

Which one of the following groups has the highest prevalence of syphilis? (check one) A. Baby boomers B. Incarcerated females C. Men who have sex with men D. Rural men 20-29 years of age E. Individuals with a history of illicit intravenous drug use

C. Men who have sex with men Factors associated with increased prevalence rates for syphilis in the United States include a history ofincarceration or commercial sex work, living in the southern or western United States, residing in a majormetropolitan area, African-American ethnicity, and being a male younger than 29 years of age. The riskfor syphilis infection is highest among men who have sex with men and among persons who areHIV-positive.The U.S. Preventive Services Task Force (USPSTF) recommends that asymptomatic, nonpregnant adultsand adolescents who are at increased risk be screened for syphilis infection (A recommendation). TheUSPSTF also recommends that local community and socioeconomic factors be considered when identifyingpatients at increased risk for infection who should be screened.

A 67-year-old male is admitted to the hospital for community-acquired pneumonia. An examination reveals a temperature of 40.0°C (104.0°F), a respiratory rate of 50/min, a pulse rate of 110 beats/min, a blood pressure of 90/50 mm Hg, and an oxygen saturation of 88% on room air. The patient is confused and requires aggressive fluid resuscitation for hypotension and he is transferred to the intensive-care unit. He has no known additional risk factors or exposures.In addition to treatment with ceftriaxone and azithromycin (Zithromax), which one of the following medications is most likely to result in improved outcomes? (check one) A. Clindamycin (Cleocin) B. Levofloxacin (Levaquin) C. Methylprednisolone (Medrol) D. Oseltamivir (Tamiflu)

C. Methylprednisolone (Medrol) This patient has severe community-acquired pneumonia based on clinical criteria, including an elevatedrespiratory rate, confusion, and hypotension requiring aggressive fluid resuscitation. Corticosteroids suchas methylprednisolone have been shown to improve clinical outcomes such as length of stay, duration ofantibiotic treatment, and the risk of developing adult respiratory distress syndrome. The preferred choiceof antibiotic treatment for patients in the intensive-care unit is a -lactam antibiotic (ceftriaxone,cefotaxime) or ampicillin/sulbactam, plus a macrolide alone or a macrolide and a respiratoryfluoroquinolone. The addition of levofloxacin is not necessarily preferred over just ceftriaxone andazithromycin. Clindamycin is not indicated in the absence of risk factors for anaerobic infection such asaspiration or alcoholism. Oseltamivir is not indicated in the absence of known or suspected influenza infection.

A 22-year-old male presents to your office the morning after falling onto his outstretched right hand as he tripped while leaving a bar. He has a deep, dull ache in the right wrist on the radial side. The pain is worsened by gripping and squeezing. On examination there is some wrist fullness and the wrist is tender to palpation over the anatomic snuffbox. Radiographs of the wrist are negative.Which one of the following would be most appropriate at this time? (check one) A. Rest, ice, compression, elevation, and NSAIDs with no specific follow-up B. Rest, ice, compression, elevation, and NSAIDs with a follow-up examination in 2 weeks C. Placement of a thumb spica splint, with a follow-up examination in 2 weeks D. CT of the wrist to detect an occult fracture E. Ultrasonography of the wrist to detect a ligament injury

C. Placement of a thumb spica splint, with a follow-up examination in 2 weeks The history, symptoms, and physical examination findings in this case suggest a scaphoid fracture. Thescaphoid bone is the most commonly fractured carpal bone and a fall on an outstretched hand can produceenough force to cause this fracture. This fracture is most common in males 15-30 years of age.The finding of anatomic snuffbox tenderness is highly sensitive but not specific for a scaphoid fracture.Initial radiographs often do not demonstrate a fracture. When there is a high clinical suspicion for ascaphoid fracture but radiographs are negative, it is reasonable to immobilize in a thumb spica splint andreevaluate in 2 weeks.

An otherwise asymptomatic 7-year-old male has a blood pressure above the 95th percentile for gender, age, and height on serial measurements. Which one of the following studies would be most appropriate at this time? (check one) A. Renin and aldosterone levels B. 24-hour urinary fractionated metanephrines and normetanephrines C. Renal ultrasonography D. Doppler ultrasonography of the renal arteries E. A sleep study

C. Renal ultrasonography Renal parenchymal diseases such as glomerulonephritis, congenital abnormalities, and reflux nephropathyare the most common cause of hypertension in preadolescent children. Preadolescent children withhypertension should be evaluated for possible secondary causes and renal ultrasonography should be thefirst choice of imaging in this age group.Renin and aldosterone levels are indicated if there is a reason to suspect primary hyperaldosteronism, suchas unexplained hypokalemia. Measurement of 24-hour urinary fractionated metanephrines andnormetanephrines is used to diagnose pheochromocytomas, which are rare and usually present with a triadof symptoms including headache, palpitations, and sweating. Doppler ultrasonography of the renal arteriesis useful for diagnosing renal artery stenosis, which should be suspected in patients with coronary orperipheral atherosclerosis or young adults, especially women 19-39 years of age, who are more at risk forrenal artery stenosis due to fibromuscular dysplasia. Sleep studies are indicated in patients who are obeseor have signs or symptoms of obstructive sleep apnea.

A 69-year-old male presents for an annual health maintenance examination. His medical history is significant for hypertension and worsening back pain over the last 6 months. Laboratory studies reveal a hemoglobin level of 8.6 g/dL (N 14.0-18.0) and a mean corpuscular volume of 88 μm3 (N 80-94). The remainder of the CBC is normal. A peripheral smear and a ferritin level are both normal. A comprehensive metabolic panel is normal except for a serum creatinine level of 1.6 mg/dL (N 0.7-1.3).Which one of the following would be the most appropriate next step in the evaluation of this patient? (check one) A. A vitamin B12 level B. A haptoglobin level C. Serum protein electrophoresis D. Flow cytometry E. A bone marrow biopsy

C. Serum protein electrophoresis This patient's laboratory results and back pain suggest multiple myeloma (MM). He has a normocyticanemia and evidence of renal insufficiency, which can indicate MM. The laboratory findings along withworsening back pain indicate a need to order serum protein electrophoresis to look for MM. Flowcytometry is generally used in patients with an elevated WBC count and suspected lymphoma. Theremainder of this patient's CBC is normal, which makes a bone marrow issue less likely. His meancorpuscular volume is also normal, making vitamin B12 deficiency less likely. A haptoglobin level couldbe ordered, but protein electrophoresis is a better choice because the peripheral smear demonstrated noevidence of a hemolytic problem.

A 69-year-old female presents to your office with a 5-day history of cough and low-grade fever. She has a past history of hypertension and obstructive sleep apnea. Her daughter brought her in this morning because of worsening symptoms. The patient's temperature is 37.4°C (99.3°F), her blood pressure is 110/74 mm Hg, her pulse rate is 88 beats/min, her respiratory rate is 36/min, and her oxygen saturation is 95% on room air. She is alert and oriented to person, place, and time. A CBC and basic metabolic panel are normal except for an elevated WBC count of 12,500/mm3 (N 4300-10,800). A chest radiograph shows a right lower lobe infiltrate. This patient has a higher risk of mortality and should be considered for inpatient treatment due to her (check one) A. female sex B. underlying hypertension C. respiratory rate D. elevated WBC count E. abnormal chest radiograph

C. respiratory rate There are several decision support tools to assist in predicting 30-day mortality for patients withcommunity-acquired pneumonia. Calculating the number of high-risk markers can aid in deciding whetherto admit the patient to the hospital. The risk of mortality increases with a respiratory rate > 30/min,hypotension, confusion or disorientation, a BUN level > 20 mg/dL, age >65 years, male sex, or the presence of heart failure or COPD.

A 67-year-old female reports hearing a ringing sound when she is in a quiet room. The ringing is not bothersome to her, but she wonders what is causing it. She has not noticed any hearing loss.According to the American Academy of Otolaryngology—Head and Neck Surgery, neurologic imaging (such as contrast-enhanced MRI of the brain) would be indicated if A. the patient requests imaging B. the tinnitus is nonpulsatile C. the tinnitus is unilateral D. treatment with an antidepressant such as fluoxetine (Prozac) fails E. audiology testing identifies symmetric, mild, high-frequency hearing loss

C. the tinnitus is unilateral Tinnitus that is bilateral and not bothersome can be treated conservatively with cognitive-behavioral therapy, sound therapy, and, if appropriate, hearing aids. Antidepressants are not recommended. Pulsatile tinnitus, unilateral tinnitus, or tinnitus associated with asymmetric hearing loss is more likely to be associated with a pathologic cause. Symmetric, mild, high-frequency hearing loss is common in elderly patients. Imaging should not be part of the routine management of tinnitus that does not have warning signs, and patients should be counseled on conservative measures as described.

A 14-year-old female is brought to your office for an annual well child check and sports preparticipation physical examination. She says she does a lot of running during basketball practices and games but has trouble controlling her weight. Most of her family is overweight. She does not have any difficulty participating in sports, and has no symptoms such as chest pain, shortness of breath, or headaches. She has no significant past medical history.On examination the patient's height is 154 cm (61 in) and she weighs 63 kg (139 lb). Her BMI is 26 kg/m2, which places her in the 90th percentile for her age. Her blood pressure is 130/85 mm Hg, which places her between the 95th and 99th percentile for her age, height, and sex. Her chart reveals that her blood pressure was at this level at the last two visits. The physical examination is otherwise normal. In addition to counseling and support for weight loss, which one of the following would be most appropriate at this point? (check one) A. Informing the patient and her parents that she is prehypertensive and having her return for a blood pressure check in 3 months B. Plasma renin and catecholamine levels C. An imaging study of the renal arteries D. A fasting basic metabolic panel, a lipid profile, and a urinalysis E. Antihypertensive drug therapy

D. A fasting basic metabolic panel, a lipid profile, and a urinalysis In a pediatric patient, blood pressure should be evaluated using comparisons based on age, sex, and height. Although this adolescent's blood pressure is prehypertensive for an adult according to JNC 8 guidelines, it is stage 1 hypertension (between 95% and 99%) for her age, sex, and height. All pediatric patients with confirmed hypertension should have further evaluation to check for renal dysfunction as well as other cardiac risk factors. Additionally, renal ultrasonography is recommended to evaluate for renal disease and echocardiography to evaluate for end-organ damage that would affect treatment goals. Additional studies, such as plasma renin and catecholamine levels or renovascular imaging, may be indicated in children with abnormalities on initial evaluation that suggest secondary causes of hypertension.Pharmacologic therapy is usually recommended for pediatric patients with symptomatic hypertension, secondary hypertension, target organ damage, diabetes mellitus, or persistent hypertension in spite of nonpharmacologic treatment. A low-sodium diet may be helpful for decreasing blood pressure, and given this patient's obesity, intensive counseling about lifestyle changes is appropriate.

Additional workup or referral to an endocrinologist for evaluation of precocious puberty would be indicated in which one of the following patients? (check one) A. A 7-year-old female with some pubic hair B. An 8-year-old female with breast buds C. An 8-year-old male with some pubic hair and axillary odor D. An 8-year-old male with penile enlargement E. A 10-year-old female who has recently begun having menses

D. An 8-year-old male with penile enlargement Penile enlargement in an 8-year-old male is a sign of precocious puberty. Isolated sparse pubic and axillary hair growth and axillary odor is referred to as premature adrenarche, and represents high levels of dehydroepiandrosterone rather than activation of the hypothalamic-pituitary-gonadal axis that leads to puberty. The isolated findings of premature adrenarche are generally considered benign. An 8-year-old with breast buds and a 10-year-old with menarche are within the normal range of expected pubertal development. Penile enlargement typically represents full activation of the hypothalamic-pituitary-gonadal axis and warrants endocrinologic evaluation in boys younger than 9 years of age.

A 50-year-old female sees you for follow-up of her hypertension. At her last visit 4 weeks ago you started her on lisinopril (Prinivil, Zestril), 10 mg daily, because of a blood pressure of 158/92 mm Hg and confirmed hypertension on ambulatory blood pressure monitoring. She is tolerating the medication well and has no side effects. She does not take any other medications. Today her blood pressure is 149/90 mm Hg, which you confirm on repeat measurement. This is also consistent with her home measurements. At her last visit a basic metabolic panel was normal.You repeat a basic metabolic panel today and the results are normal except for a BUN of 25 mg/dL (N 8-23) and a creatinine level of 1.5 mg/dL (N 0.6-1.1). At her last visit her BUN was 12 mg/dL and her creatinine level was 0.7 mg/dL.Which one of the following would be most appropriate at this time? (check one) A. Continue her current treatment regimen B. Increase lisinopril to 20 mg daily C. Continue lisinopril at the current dosage and add amlodipine (Norvasc), 5 mg daily D. Discontinue lisinopril and begin amlodipine, 5 mg daily E. Discontinue lisinopril and begin losartan (Cozaar), 25 mg daily

D. Discontinue lisinopril and begin amlodipine, 5 mg daily This patient has essential hypertension and her goal blood pressure is <140/90 mm Hg based on JNC 8 guidelines, or 130/80 mm Hg based on the more recent recommendations of the American College of Cardiology/American Heart Association Task Force on Clinical Practice Guidelines. Until recently, it was recommended that physicians should tolerate a rise of <30% in serum creatinine after ACE inhibitor or angiotensin receptor blocker (ARB) initiation. Rises in serum creatinine of >30% from baseline increase the risk of renal failure, adverse cardiac outcomes, and death. A recent study suggests that rises in serum creatinine of <30% also put patients at risk for these outcomes, with a dose-response relationship between the magnitude of creatinine change and the risk of adverse outcomes.This patient has more than a 30% rise in creatinine and has no other factors, such as diabetes mellitus, heart failure, or chronic kidney disease, that would indicate a need for ACE or ARB therapy for her hypertension. Discontinuing her ACE inhibitor and starting a medication from a different class is the most appropriate treatment at this time. Based on JNC 8 guidelines, additional options for blood pressure medications include thiazide diuretics and calcium channel blockers.

A 24-year-old male presents with swelling of the right testicle. The pain started yesterday and has persistently worsened. A physical examination reveals swelling on the right side of his scrotum. His entire testicle is painful, and elevation of the scrotum improves his pain. Ultrasonography reveals hyperemia, swelling, and increased blood flow to the testis and epididymis.Which one of the following would be the most appropriate management? (check one) A. Doxycycline B. Levofloxacin (Levaquin) C. Trimethoprim/sulfamethoxazole (Bactrim) D. Doxycycline plus ceftriaxone E. Urgent surgical evaluation

D. Doxycycline plus ceftriaxone This patient presents with acute epididymitis. Typical symptoms develop gradually over 1-2 days withposterior scrotal pain and swelling. Additional symptoms may include fever, hematuria, dysuria, andurinary frequency. The pain may radiate to the lower abdomen. Physical examination findings may includetenderness of the epididymis and testis along with swelling of the scrotum. Elevation of the scrotum maydecrease the pain (Prehn sign). Typical ultrasound findings include hyperemia, swelling, and increasedblood flow to the epididymis.With testicular torsion the pain is often sudden in onset and severe, with associated nausea and vomitingand no other urologic symptoms. A physical examination often demonstrates a high-riding testis that maylie transversely in the scrotum. The cremasteric reflex may be absent. Ultrasound findings woulddemonstrate decreased or absent blood flow with testicular torsion.In sexually active adults <35 years of age, gonorrhea and Chlamydia are the most common causativeorganisms of acute epididymitis. Ceftriaxone, 250 mg intramuscularly or intravenously once, with oraldoxycycline, 100 mg twice daily for 10 days, would be the appropriate treatment for acute epididymitis(SOR C). In men over the age of 35 or those with a history of recent urinary tract surgery orinstrumentation, enteric organisms are the most likely cause and monotherapy with oral levofloxacin orofloxacin for 10 days would be the recommended treatment.

A 68-year-old female with diabetes mellitus, coronary artery disease, fibromyalgia, anddyspepsia presents for follow-up. She has been taking omeprazole (Prilosec) for 10 years. It wasstarted during a hospitalization, and her symptoms have returned with previous trials ofdiscontinuation.Which one of the following adverse events is this patient at risk for as a result of her omeprazole use? (check one) A. Hypermagnesemia B. Urinary tract infections C. Nephrolithiasis D. Hip fractures

D. Hip fractures Proton pump inhibitors (PPIs), including omeprazole, are generally safe and effective for peptic ulcerdisease, gastroesophageal reflux disease, and stress ulcer prevention in critically ill patients. As use hasincreased, however, risks of long-term use of PPIs have emerged. Currently known risks include increasedfractures of the hip, wrist, and spine (SOR B), community-acquired pneumonia (SOR B), Clostridiumdifficile and other enteric infections (SOR C), hypomagnesemia (SOR B), and cardiac events whencoadministered with clopidogrel (SOR B). PPIs may also affect the absorption of vitamins and minerals,including iron, vitamin B12, and folate (SOR C). There is no known association of PPIs with nephrolithiasisor urinary tract infections.

A 19-year-old female member of a college cross-country team presents with a 1-week history of right knee pain. She does not have any acute injury to the knee. An examination reveals no deformity and she has a normal gait. She has tenderness and subtle swelling localized 1 cm distal to the right medial joint line, and examinations of the knee and hip are otherwise normal.Which one of the following is the most likely diagnosis? (check one) A. Fibular head stress fracture B. Iliotibial band syndrome C. Medial meniscal tear D. Pes anserine bursitis E. Tibial apophysitis (Osgood-Schlatter disease)

D. Pes anserine bursitis This patient has medial knee pain related to repetitive use, most likely caused by pes anserine bursitis.Iliotibial band syndrome is often related to overuse but causes pain in the lateral knee. The fibular headis also lateral to the knee joint. Osgood-Schlatter disease is also often related to overuse but causes painat the insertion of the patellar ligament on the midline proximal tibia. A medial meniscal tear wouldlocalize to the medial joint line rather than distal to the joint line and would more likely be associated withpositive findings from other examinations, such as a McMurray test.

A 68-year-old female presents with a history of episodic severe lower abdominal pain relieved by defecation. She has had a long history of constipation with normal to very firm stools. Her history and a physical examination are otherwise normal. A colonoscopy 3 years ago was normal. You diagnose constipation-predominant irritable bowel syndrome.Which one of the following agents would be the most appropriate treatment for this patient? What is the best initial management for this patient? (check one) A. Lactulose B. Magnesium citrate C. Milk of magnesia D. Polyethylene glycol E. Sodium phosphate

D. Polyethylene glycol Hypertonic osmotic laxatives such as milk of magnesia, magnesium citrate, and sodium phosphate draw water into the bowel and should be used with caution in older adults and those with renal impairment because of the risk of electrolyte abnormalities and dehydration in patients with irritable bowel syndrome (IBS). Lactulose, also an osmotic laxative, should be avoided in patients with IBS because it is broken down by colonic flora and produces excessive gas. Polyethylene glycol, a long-chain polymer of ethylene oxide, is a large molecule that causes water to be retained in the colon, which softens the stool and increases the number of bowel movements. It is approved by the FDA for short-term treatment in adults and children with occasional constipation and is commonly prescribed for patients with IBS. It is considered safe and effective for moderate to severe constipation when used either daily or as needed.

In the elderly, the risk of heat wave-related death is highest in those who (check one) A. have COPD B. have diabetes and are insulin-dependent C. have a functioning fan, but not air conditioning D. are homebound

D. are homebound

A 30-year-old female presents for follow-up after an emergency department visit for an episode of symptomatic supraventricular tachycardia that was diagnosed as Wolff-Parkinson-White syndrome. Which one of the following would be most appropriate for the initial long-term management of this patient? (check one) A. Adenosine (Adenocard) B. Amiodarone (Cordarone) C. Diltiazem (Cardizem) D. Metoprolol E. Catheter ablation

E. Catheter ablation Catheter ablation is the most appropriate treatment for a patient with symptomatic Wolff-Parkinson-Whitesyndrome (WPW). Catheter ablation has a very high immediate success rate (96%-98%). The mostsignificant risk associated with the procedure is permanent atrioventricular block, which occurs inapproximately 0.4% of procedures. Adenosine and amiodarone are used for the acute management ofsupraventricular tachycardia, but not for long-term management. Node-blocking medications such asdiltiazem and metoprolol should not be used for the long-term treatment of WPW, due to the increased riskof ventricular fibrillation.

A patient is admitted to the hospital with severe acute pancreatitis, based on diagnostic criteria for severity. After appropriate intravenous hydration, which one of the following is associated with shorter hospital stays and lower mortality? (check one) A. Parenteral nutrition B. Nothing by mouth until the pain has resolved C. Clear liquids by mouth after 48 hours D. Bolus nasogastric enteral nutrition E. Continuous nasogastric enteral nutrition

E. Continuous nasogastric enteral nutrition

A 45-year-old female presents with pain in her right hand and wrist. She is right hand-dominant and most symptoms are in the right hand and wrist. She has been awakening at night with numbness of the hand that improves after she shakes her wrist. Her work as a software engineer involves lots of typing.On examination you note a positive Tinel's sign at the right wrist consistent with carpal tunnel syndrome. She has an important project at work over the next several months and will not be able to take any time off.Which one of the following treatments has the best evidence for delaying the need for definitive surgical therapy? (check one) A. Night splints B. Physical therapy C. Therapeutic ultrasound D. Oral corticosteroids E. Corticosteroid injection

E. Corticosteroid injection

A 64-year-old male with midsternal chest pain is brought to the emergency department by ambulance. He is on oxygen and an intravenous line is in place. Shortly after arrival he loses consciousness and becomes pulseless and apneic, and CPR is begun. Cardiac monitoring shows ventricular tachycardia with a rate of 160 beats/min.Which one of the following would be most appropriate at this point? (check one) A. Amiodarone, intravenous infusion, followed by synchronized cardioversion B. Adenosine (Adenocard), rapid intravenous push, repeated in 1-2 minutes if needed C. Epinephrine, intravenous push, followed by synchronized cardioversion D. Lidocaine (Xylocaine), intravenous push, repeated in 5 minutes if needed E. Defibrillation

E. Defibrillation Pulseless ventricular tachycardia (VT) should be treated the same as ventricular fibrillation. The first step is defibrillation. If that is unsuccessful, epinephrine is administered and defibrillation is reattempted. Lidocaine, adenosine, and procainamide may be used for the initial treatment of a wide-complex tachycardia of uncertain type, but should not be used for the initial treatment of pulseless VT. Synchronized cardioversion alone would be indicated for the initial treatment of rapid unstable tachycardia with a pulse.

In older patients with aortic stenosis and a systolic murmur, which one of the following wouldbe most concerning? (check one) A. Weight loss B. Frequent urination C. Jaundice D. Worsening headache E. Exertional dyspnea

E. Exertional dyspnea

A patient comes to your outpatient clinic with a persistent migraine that she has been unable to treat effectively at home. The symptoms began several hours ago and are typical for her. She has already tried her usual treatments of ibuprofen, 800 mg, and rizatriptan (Maxalt), 10 mg, but they have not provided any relief. She took a second dose of rizatriptan 2 hours later without benefit. She is in significant pain, which is causing mild nausea, and she has photophobia and phonophobia.Which one of the following would be most appropriate at this point? A. Oral butalbital/acetaminophen/caffeine (Fioricet) B. Oral ergotamine/caffeine (Cafergot) C. Subcutaneous sumatriptan (Imitrex) D. Intramuscular morphine E. Intramuscular prochlorperazine

E. Intramuscular prochlorperazine Multiple studies have determined that parenteral antiemetics have benefits for the treatment of acutemigraine beyond their effect on nausea. Most outpatient clinics do not have the ability to administerintravenous metoclopramide, which is the preferred treatment. However, most clinics do have the abilityto administer intramuscular prochlorperazine or promethazine. Due to concerns about oversedation,misuse, and rebound, treatment with parenteral opiates is discouraged but may be an option if othertreatments fail. Oral butalbital/acetaminophen/caffeine and oral ergotamine/caffeine have less evidence ofsuccess in the treatment of acute migraine. Sumatriptan is contraindicated within 24 hours of the use ofrizatriptan.

A 48-year-old male with an 8-week history of the gradual onset of nonradiating, worsening left heel pain sees you for follow-up. He started running on his neighborhood streets 3 months ago to train for a 10K race. The pain limits his training significantly. His symptoms improved when he began taking ibuprofen and took 2 weeks off from running a month ago. A radiograph of the left foot 4 weeks ago was normal. There is no other pain and he feels well otherwise.On examination you note that his left heel is slightly swollen compared to the right and very tender when squeezed on the sides. The anterior aspect and Achilles tendon insertion of the heel are nontender. There is no erythema or warmth and the remainder of the left lower extremity examination is normal. His vital signs are normal.Which one of the following would you recommend at this point? (check one) A. A C-reactive protein level B. Nerve conduction velocity testing C. A repeat radiograph D. Ultrasonography E. MRI

E. MRI This patient has a calcaneal stress fracture as suggested by the history of increased running on a hardsurface, improvement with rest, and a positive calcaneal squeeze on examination. A delay in diagnosisincreases the risk of delayed union. MRI is the preferred imaging modality because radiographs often donot detect a calcaneal stress fracture. A C-reactive protein level could be indicated if there were symptomsor signs of infection or autoimmune illness. The clinical picture does not suggest a neurologic condition,so nerve conduction velocity testing is not appropriate. While there are some case reports of the diagnosisof stress fractures using ultrasonography, this is not the preferred imaging method.

A 60-year-old male with a long-standing history of hypertension seeks your advice about painrelief from his osteoarthritis. He has tried acetaminophen and topical capsaicin cream withoutmuch benefit. He is concerned about media reports of NSAIDs causing heart problems and isunsure which ones would be safest for him to use.Based on current evidence, which one of the following NSAIDs would you recommend as beingLEAST likely to be associated with an increased risk of myocardial infarction? (check one) A. Celecoxib (Celebrex) B. Diclofenac (Zorvolex) C. Ibuprofen D. Meloxicam (Mobic) E. Naproxen (Naprosyn)

E. Naproxen (Naprosyn)

A 78-year-old female presents with a red eye. She reports drainage and pain in her left eye sinceshe woke up today, but no photophobia. Examination of the eye shows conjunctival erythemaand a mucopurulent discharge. The pupil is normal in size and reactive to light.Which one of the following should prompt immediate referral to an ophthalmologist? A. Bilateral eye redness B. A corneal abrasion noted on fluorescein staining C. Copious mucopurulent drainage from the eye D. Bright red blood noted under the conjunctiva E. Reduction of visual acuity

E. Reduction of visual acuity Reduced visual acuity may be a symptom of acute angle-closure glaucoma and requires immediate referralto an ophthalmologist. Copious mucopurulent drainage from the eye is a sign of infectious conjunctivitis,most likely bacterial, and bilateral eye redness is typically seen with allergic conjunctivitis. Allergic orinfectious conjunctivitis and small corneal abrasions can be managed by the family physician. Bright redblood under the conjunctiva is consistent with a subconjunctival hemorrhage that will typically resolvewithout intervention.

A 50-year-old male presents with difficulty straightening his left ring finger. Examination of the affected hand reveals a nodule of the palmar aponeurosis and associated fibrous band that limits full extension of the fourth finger. He is unable to fully extend both the metacarpophalangeal (MCP) joint and the proximal interphalangeal (PIP) joint, with MCP and PIP contractures estimated at 40° and 20°, respectively. Which one of the following would be the most appropriate management strategy? (check one) A. Observation until the PIP contracture is >90° B. Serial intralesional injection with a corticosteroid C. Cryosurgery of the fibrous nodule D. Referral for physical therapy E. Referral for surgical release of the contracture

E. Referral for surgical release of the contracture This patient has Dupuytren's disease with a contracture of the affected finger. Surgical release is indicatedwhen the metacarpophalangeal joint contracture reaches 30° or with any degree of contracture of theproximal interphalangeal joint. Intralesional injection may reduce the need for later surgery in a patientwith grade 1 disease, but not if there is a contracture. There is no evidence to support the use of physicaltherapy or cryosurgery.

Which one of the following community health programs best fits the definition of secondaryprevention? (check one) A. An antismoking education program at a local middle school B. Blood pressure screening at a local church C. A condom distribution program D. Screening patients with diabetes mellitus for microalbuminuria

Prevention traditionally has been divided into three categories: primary, secondary, and tertiary. Primary prevention targets individuals who may be at risk to develop a medical condition and intervenes to prevent the onset of that condition. Examples include childhood vaccination programs, water fluoridation, antismoking programs, and education about safe sex. Secondary prevention targets individuals who have developed an asymptomatic disease and institutes treatment to prevent complications. Examples include routine Papanicolaou tests and screening for hypertension, diabetes mellitus, or hyperlipidemia. Tertiary prevention targets individuals with a known disease, with the goal of limiting or preventing future complications. Examples include screening patients with diabetes for microalbuminuria, rigorous treatment of diabetes mellitus, and post-myocardial infarction prophylaxis with β-blockers and aspirin.

Metformin should not be used in patients with a a bad GFR T/f

T

A 43-year-old female presents to your office with a 3-month history of left low back and posterior hip pain. She does not recall an injury but says she was very active during a move to a new home prior to the onset of the pain. An examination reveals that her gait, lower extremity strength, straight leg resistance, and hip and knee range of motion are normal. A log roll test is also normal. A flexion, abduction, external rotation (FABER) test produces posterior pain. Which one of the following is the most likely diagnosis? (check one) A. Femoroacetabular impingement B. Greater trochanteric pain syndrome C. Osteoarthritis D. Piriformis syndrome E. Sacroiliac joint dysfunction

E. Sacroiliac joint dysfunction The cause of hip pain is generally determined from the patient's history and physical examination. Apositive flexion, abduction, external rotation (FABER) test that produces pain at the sacroiliac joint, lumbarspine, and posterior hip is associated with sacroiliac joint dysfunction. The log roll test involves passivesupine internal and external rotation of the hip. When this test is positive for pain it is associated withpiriformis syndrome. While femoroacetabular impingement may be associated with a positive FABER test,it would produce pain in the groin. Greater trochanteric pain syndrome results in lateral hip pain ratherthan posterior pain. Osteoarthritis is usually associated with a limited range of motion and groin pain.

Which one of the following antihypertensive drugs may reduce the severity of sleep apnea? (check one) A. Amlodipine (Norvasc) B. Hydralazine C. Lisinopril (Prinivil, Zestril) D. Metoprolol E. Spironolactone (Aldactone)

E. Spironolactone (Aldactone)

A 46-year-old female with a history of hyperthyroidism controlled with methimazole (Tapazole), 10 mg daily, returns to your office after an absence of several years. She has new symptoms of palpitations, heat intolerance, and hoarseness. A physical examination reveals an enlarged thyroid and a radioactive iodine uptake scan shows accumulation of tracer in multiple areas. Which one of the following is the appropriate definitive treatment for this patient? (check one) A. Methimazole alone, 20 mg daily B. Methimazole, 20 mg daily, plus propranolol, 80 mg twice daily C. Propylthiouracil alone, 50 mg 3 times daily D. Radioactive iodine E. Thyroidectomy

E. Thyroidectomy This patient has a medical history, physical examination, and radioactive iodine uptake scan consistent withtoxic multinodular goiter, which is the second most common cause of hyperthyroidism in the United States.Although the addition of propranolol and an increase in methimazole may control her palpitations and othersymptoms of hyperthyroidism, these measures will not permanently eliminate the problem. Radioactiveiodine ablation and thyroidectomy with subsequent thyroid hormone replacement are both appropriatetreatments for toxic multinodular goiter, but thyroidectomy is indicated for this patient because she hascompressive symptoms from the goiter itself.

In patients with COPD, which one of the following inhaled medications has been shown to reduce exacerbations and exacerbation-related hospitalizations? (check one) A. Albuterol (Proventil, Ventolin) B. Fluticasone (Flovent) C. Ipratropium (Atrovent) D. Salmeterol (Serevent) E. Tiotropium (Spiriva)

E. Tiotropium (Spiriva) A Cochrane review found that the long-acting antimuscarinic agent tiotropium improved quality of life and reduced exacerbations and exacerbation-related hospitalizations in patients with underlying COPD. Tiotropium was noted to be superior to long-acting β-agonists such as salmeterol. Albuterol, fluticasone, and ipratropium have not been shown to have these effects (SOR A).

A 2-year-old African-American male with a history of sickle cell disease is brought to your office for a well child check. Which one of the following would be most appropriate for screening at this time? (check one) A. A chest radiograph B. A DXA scan C. Abdominal ultrasonography D. Renal Doppler ultrasonography E. Transcranial Doppler ultrasonography

E. Transcranial Doppler ultrasonography

A 45-year-old male sees you for follow-up after a pre-employment physical examination reveals blood in his urine. He brings a copy of a urinalysis report that shows 3-5 RBCs/hpf. He has not seen any gross blood himself. He is asymptomatic, is on no medications, and does not smoke. You perform a physical examination, with normal findings. A repeat urinalysis confirms the presence of red blood cells but is otherwise normal. Which one of the following would be most appropriate at this point? (check one) A. Observation and reassurance B. A repeat urinalysis in 6 months C. Urine cytology only D. Ultrasonography of the kidneys and urine cytology only E. Ultrasonography of the kidneys, urine cytology, and cystoscopy

E. Ultrasonography of the kidneys, urine cytology, and cystoscopy

A 38-year-old white female presents with abdominal pain and insists that she be referred for surgical evaluation. She has a history of multiple unexplained physical symptoms that began in her late teenage years. She is vague about past medical evaluations, but a review of her extensive medical record reveals multiple normal blood and imaging tests, several surgical procedures that have failed to alleviate her symptoms, and frequent requests for refills of narcotic analgesics. This patient's history is most compatible with (check one) A. illness anxiety disorder B. malingering C. panic disorder D. generalized anxiety disorder E. somatic symptom disorder

E. somatic symptom disorder Somatic symptom disorder (formerly called somatization disorder) usually begins in the teens or twenties and is characterized by multiple unexplained physical symptoms, insistence on surgical procedures, and an imprecise or inaccurate medical history. These patients also commonly abuse alcohol, narcotics, or other drugs.Patients with illness anxiety disorder are overly concerned with bodily functions, and can often provide accurate, extensive, and detailed medical histories. Malingering is an intentional pretense of illness to obtain personal gain. Patients with panic disorder have episodes of intense, short-lived attacks of cardiovascular, neurologic, or gastrointestinal symptoms. Generalized anxiety disorder is characterized by unrealistic worry about life circumstances accompanied by symptoms of motor tension, autonomic hyperactivity, or vigilance and scanning.

Cluster headaches

Orbital area , can occur within 2-3 hours of falling asleep

best predictor of severity of acute pancreatitis ?

hct

Primary prevention examples

immunizations, pollution control, antismoking programs, and education about safe sex

A history of an _____________ _____________ within the past 3 months is an absolute contraindication to fibrinolytic therapy in patients with an ST-elevation myocardial infarction (STEMI), unless the stroke is diagnosed within 4½ hours

ischemic stroke

Secondary prevention examples

routine Papanicolaou smears, and screening for hypertension, diabetes, or hyperlipidemia


Set pelajaran terkait

13. Can You Help Me Find Something?

View Set

Chapter 9: Firms in Competitive Markets: The Long Run

View Set

Law and Legal Profession Review 3

View Set

Chapter 24 Gastrointestinal Chapter 27 Genitourinary

View Set

More PrepU ch 26 Emergency cards

View Set

7.7 Arrays of Objects, 7.8 The Sequential Search Algorithm

View Set

EVOLVE REACH (HESI) A2 PRACTICE TEST - MODULE 4

View Set

Domain 1: Security and Risk Management: Business Continuity

View Set

Lewis Med-Surg Ch. 13 Altered Immune Responses and Transplantation

View Set